Prove that the maximum of $n$ independent standard normal random variables, is asymptotically equivalent to...












10












$begingroup$


Lets $(X_n)_{ninmathbb{N}}$ be an iid sequence of standard normal random variables. Define $$M_n=max_{1leq ileq n} X_i.$$
Prove that $$lim_{nrightarrowinfty} frac{M_n}{sqrt{2log n}}=1quadtext{a.s.}$$



I used the fact that $$left(frac{1}{x}-frac{1}{x^3}right)e^{-frac{x^2}{2}}leqmathbb P(X_n>x)leq frac{1}{x}e^{-frac{x^2}{2}},$$



and the Borel Cantelli lemmas to prove that
$$limsup_{nrightarrowinfty} frac{X_n}{sqrt{2log n}}=1quadtext{a.s.}$$



I used Davide Giraudo's comment to show $$limsup_n frac{M_n}{sqrt{2log n}}=1quad text{a.s.}$$



I have no idea how to compute the $liminf$. Borel-Cantelli give us tools to compute the $limsup$ of sets, I am unsure of how to argue almost sure convergence. Any help would be appreciated.










share|cite|improve this question











$endgroup$

















    10












    $begingroup$


    Lets $(X_n)_{ninmathbb{N}}$ be an iid sequence of standard normal random variables. Define $$M_n=max_{1leq ileq n} X_i.$$
    Prove that $$lim_{nrightarrowinfty} frac{M_n}{sqrt{2log n}}=1quadtext{a.s.}$$



    I used the fact that $$left(frac{1}{x}-frac{1}{x^3}right)e^{-frac{x^2}{2}}leqmathbb P(X_n>x)leq frac{1}{x}e^{-frac{x^2}{2}},$$



    and the Borel Cantelli lemmas to prove that
    $$limsup_{nrightarrowinfty} frac{X_n}{sqrt{2log n}}=1quadtext{a.s.}$$



    I used Davide Giraudo's comment to show $$limsup_n frac{M_n}{sqrt{2log n}}=1quad text{a.s.}$$



    I have no idea how to compute the $liminf$. Borel-Cantelli give us tools to compute the $limsup$ of sets, I am unsure of how to argue almost sure convergence. Any help would be appreciated.










    share|cite|improve this question











    $endgroup$















      10












      10








      10


      9



      $begingroup$


      Lets $(X_n)_{ninmathbb{N}}$ be an iid sequence of standard normal random variables. Define $$M_n=max_{1leq ileq n} X_i.$$
      Prove that $$lim_{nrightarrowinfty} frac{M_n}{sqrt{2log n}}=1quadtext{a.s.}$$



      I used the fact that $$left(frac{1}{x}-frac{1}{x^3}right)e^{-frac{x^2}{2}}leqmathbb P(X_n>x)leq frac{1}{x}e^{-frac{x^2}{2}},$$



      and the Borel Cantelli lemmas to prove that
      $$limsup_{nrightarrowinfty} frac{X_n}{sqrt{2log n}}=1quadtext{a.s.}$$



      I used Davide Giraudo's comment to show $$limsup_n frac{M_n}{sqrt{2log n}}=1quad text{a.s.}$$



      I have no idea how to compute the $liminf$. Borel-Cantelli give us tools to compute the $limsup$ of sets, I am unsure of how to argue almost sure convergence. Any help would be appreciated.










      share|cite|improve this question











      $endgroup$




      Lets $(X_n)_{ninmathbb{N}}$ be an iid sequence of standard normal random variables. Define $$M_n=max_{1leq ileq n} X_i.$$
      Prove that $$lim_{nrightarrowinfty} frac{M_n}{sqrt{2log n}}=1quadtext{a.s.}$$



      I used the fact that $$left(frac{1}{x}-frac{1}{x^3}right)e^{-frac{x^2}{2}}leqmathbb P(X_n>x)leq frac{1}{x}e^{-frac{x^2}{2}},$$



      and the Borel Cantelli lemmas to prove that
      $$limsup_{nrightarrowinfty} frac{X_n}{sqrt{2log n}}=1quadtext{a.s.}$$



      I used Davide Giraudo's comment to show $$limsup_n frac{M_n}{sqrt{2log n}}=1quad text{a.s.}$$



      I have no idea how to compute the $liminf$. Borel-Cantelli give us tools to compute the $limsup$ of sets, I am unsure of how to argue almost sure convergence. Any help would be appreciated.







      probability probability-theory probability-distributions probability-limit-theorems






      share|cite|improve this question















      share|cite|improve this question













      share|cite|improve this question




      share|cite|improve this question








      edited Aug 6 '16 at 19:03









      Michael Hardy

      1




      1










      asked Oct 7 '14 at 3:32









      anonymousanonymous

      1206




      1206






















          2 Answers
          2






          active

          oldest

          votes


















          6












          $begingroup$

          Your estimate gives that for each positive $varepsilon$, we have
          $$sum_imathbb P(M_{2^{i}}>(1+varepsilon)sqrt 2sqrt{i+1})<infty$$
          (we use the fact that $mathbb P(M_n>x)leqslant nmathbb P(X_1gt x)$).



          We thus deduce that $limsup_iM_{2^{i}}/(sqrt{2(i+1)})leqslant 1+varepsilon$ almost surely. Taking $varepsilon:=1/k$, we get $limsup_iM_{2^{i}}/(sqrt{2(i+1)})leqslant 1$ almost surely. To conclude, notice that if $2^ileqslant nlt 2^{i+1}$,
          $$frac{M_n}{sqrt{2log n}}leqslant frac{M_{2^{i+1}}}{sqrt{2i}}.$$



          For the $liminf$, define for a fixed positive $varepsilon$, $$A_n:=left{frac{M_n}{sqrt{2log n}}lt 1-varepsilonright}.$$
          A use of the estimate on the tail of the normal distribution show that the series $sum_n mathbb P(A_n)$ is convergent, hence by the Borel-Cantelli lemma, we have $mathbb Pleft(limsup_n A_nright)=0$. This means that for almost every $omega$, we can find $N=N(omega)$ such that if $ngeqslant N(omega)$, then $$frac{M_n(omega)}{sqrt{2log n}}geqslant 1-varepsilon.$$
          This implies that $$liminf_{nto infty}frac{M_n(omega)}{sqrt{2log n}}geqslant 1-varepsilon quad mbox{a.e.}$$
          Here again, taking $varepsilon:=1/k$, we obtain that
          $$liminf_{nto infty}frac{M_n(omega)}{sqrt{2log n}}geqslant 1quad mbox{a.e.}$$






          share|cite|improve this answer











          $endgroup$













          • $begingroup$
            So I am mainly having trouble with justifying the first sum converges. I get $$mathbb{P}(M_{2^i}>sqrt{(1+epsilon)2(i+1)})=1-(1-mathbb{P}(X_{2^i}>sqrt{(1+epsilon)2(i+1)}))^{2^i} leq 1-(1-frac{1}{sqrt{(1+epsilon)2(i+1)}}e^{-(1+epsilon)(i+1)})^{2^i}$$
            $endgroup$
            – anonymous
            Oct 7 '14 at 10:15












          • $begingroup$
            I edited. Is it clearer?
            $endgroup$
            – Davide Giraudo
            Oct 7 '14 at 10:18










          • $begingroup$
            yes significantly. Thank you. So that shows that $limsup$ of $frac{M_n}{sqrt{2log n}}leq 1$, but not the $liminfgeq1$.
            $endgroup$
            – anonymous
            Oct 7 '14 at 10:27












          • $begingroup$
            It may be not the case that $liminfgeq 1$, however the limsup is greater than $1$ because $M_ngeq X_n$.
            $endgroup$
            – Davide Giraudo
            Oct 7 '14 at 11:10










          • $begingroup$
            Sorry, I missed the fact that you have a limit and not only a limsup.
            $endgroup$
            – Davide Giraudo
            Oct 7 '14 at 11:22



















          2












          $begingroup$

          Not an answer, but a related comment that is too long for a comment box ...



          This question made me curious to compare:




          • the pdf of the sample maximum (given a sample of size $n$ drawn on a N(0,1) parent), say $f(x;n):$


          $$f(x) = frac{2^{frac{1}{2}-n} n e^{-frac{x^2}{2}} left(1+text{erf}left(frac{x}{sqrt{2}}right)right)^{n-1}}{sqrt{pi }}$$
          where erf(.) denotes the error function, to




          • the asymptote proposed by the question $sqrt{2 log n}$


          ... when $n$ is very large (say $n$ = 1 million).



          The diagram compares:



          enter image description here



          The distribution does not, in fact, converge to the asymptote in any meaningful manner, even when $n$ is very large, such as $n$ = 1 million or 100 million or even a billion.






          share|cite|improve this answer











          $endgroup$













          • $begingroup$
            Are you using the right logarithm? (I.e. base $e$ and not base 10?)
            $endgroup$
            – Chill2Macht
            Oct 1 '18 at 0:13






          • 1




            $begingroup$
            Yes - $sqrt{2 log left(10^6right)}$ = 5.25 ... as shown in the diagram above (the red line). By contrast, with base 10, the same calculation would be equal to $2 sqrt{3}$ which is approx 3.46.
            $endgroup$
            – wolfies
            Oct 1 '18 at 7:17














          Your Answer





          StackExchange.ifUsing("editor", function () {
          return StackExchange.using("mathjaxEditing", function () {
          StackExchange.MarkdownEditor.creationCallbacks.add(function (editor, postfix) {
          StackExchange.mathjaxEditing.prepareWmdForMathJax(editor, postfix, [["$", "$"], ["\\(","\\)"]]);
          });
          });
          }, "mathjax-editing");

          StackExchange.ready(function() {
          var channelOptions = {
          tags: "".split(" "),
          id: "69"
          };
          initTagRenderer("".split(" "), "".split(" "), channelOptions);

          StackExchange.using("externalEditor", function() {
          // Have to fire editor after snippets, if snippets enabled
          if (StackExchange.settings.snippets.snippetsEnabled) {
          StackExchange.using("snippets", function() {
          createEditor();
          });
          }
          else {
          createEditor();
          }
          });

          function createEditor() {
          StackExchange.prepareEditor({
          heartbeatType: 'answer',
          autoActivateHeartbeat: false,
          convertImagesToLinks: true,
          noModals: true,
          showLowRepImageUploadWarning: true,
          reputationToPostImages: 10,
          bindNavPrevention: true,
          postfix: "",
          imageUploader: {
          brandingHtml: "Powered by u003ca class="icon-imgur-white" href="https://imgur.com/"u003eu003c/au003e",
          contentPolicyHtml: "User contributions licensed under u003ca href="https://creativecommons.org/licenses/by-sa/3.0/"u003ecc by-sa 3.0 with attribution requiredu003c/au003e u003ca href="https://stackoverflow.com/legal/content-policy"u003e(content policy)u003c/au003e",
          allowUrls: true
          },
          noCode: true, onDemand: true,
          discardSelector: ".discard-answer"
          ,immediatelyShowMarkdownHelp:true
          });


          }
          });














          draft saved

          draft discarded


















          StackExchange.ready(
          function () {
          StackExchange.openid.initPostLogin('.new-post-login', 'https%3a%2f%2fmath.stackexchange.com%2fquestions%2f961780%2fprove-that-the-maximum-of-n-independent-standard-normal-random-variables-is-a%23new-answer', 'question_page');
          }
          );

          Post as a guest















          Required, but never shown

























          2 Answers
          2






          active

          oldest

          votes








          2 Answers
          2






          active

          oldest

          votes









          active

          oldest

          votes






          active

          oldest

          votes









          6












          $begingroup$

          Your estimate gives that for each positive $varepsilon$, we have
          $$sum_imathbb P(M_{2^{i}}>(1+varepsilon)sqrt 2sqrt{i+1})<infty$$
          (we use the fact that $mathbb P(M_n>x)leqslant nmathbb P(X_1gt x)$).



          We thus deduce that $limsup_iM_{2^{i}}/(sqrt{2(i+1)})leqslant 1+varepsilon$ almost surely. Taking $varepsilon:=1/k$, we get $limsup_iM_{2^{i}}/(sqrt{2(i+1)})leqslant 1$ almost surely. To conclude, notice that if $2^ileqslant nlt 2^{i+1}$,
          $$frac{M_n}{sqrt{2log n}}leqslant frac{M_{2^{i+1}}}{sqrt{2i}}.$$



          For the $liminf$, define for a fixed positive $varepsilon$, $$A_n:=left{frac{M_n}{sqrt{2log n}}lt 1-varepsilonright}.$$
          A use of the estimate on the tail of the normal distribution show that the series $sum_n mathbb P(A_n)$ is convergent, hence by the Borel-Cantelli lemma, we have $mathbb Pleft(limsup_n A_nright)=0$. This means that for almost every $omega$, we can find $N=N(omega)$ such that if $ngeqslant N(omega)$, then $$frac{M_n(omega)}{sqrt{2log n}}geqslant 1-varepsilon.$$
          This implies that $$liminf_{nto infty}frac{M_n(omega)}{sqrt{2log n}}geqslant 1-varepsilon quad mbox{a.e.}$$
          Here again, taking $varepsilon:=1/k$, we obtain that
          $$liminf_{nto infty}frac{M_n(omega)}{sqrt{2log n}}geqslant 1quad mbox{a.e.}$$






          share|cite|improve this answer











          $endgroup$













          • $begingroup$
            So I am mainly having trouble with justifying the first sum converges. I get $$mathbb{P}(M_{2^i}>sqrt{(1+epsilon)2(i+1)})=1-(1-mathbb{P}(X_{2^i}>sqrt{(1+epsilon)2(i+1)}))^{2^i} leq 1-(1-frac{1}{sqrt{(1+epsilon)2(i+1)}}e^{-(1+epsilon)(i+1)})^{2^i}$$
            $endgroup$
            – anonymous
            Oct 7 '14 at 10:15












          • $begingroup$
            I edited. Is it clearer?
            $endgroup$
            – Davide Giraudo
            Oct 7 '14 at 10:18










          • $begingroup$
            yes significantly. Thank you. So that shows that $limsup$ of $frac{M_n}{sqrt{2log n}}leq 1$, but not the $liminfgeq1$.
            $endgroup$
            – anonymous
            Oct 7 '14 at 10:27












          • $begingroup$
            It may be not the case that $liminfgeq 1$, however the limsup is greater than $1$ because $M_ngeq X_n$.
            $endgroup$
            – Davide Giraudo
            Oct 7 '14 at 11:10










          • $begingroup$
            Sorry, I missed the fact that you have a limit and not only a limsup.
            $endgroup$
            – Davide Giraudo
            Oct 7 '14 at 11:22
















          6












          $begingroup$

          Your estimate gives that for each positive $varepsilon$, we have
          $$sum_imathbb P(M_{2^{i}}>(1+varepsilon)sqrt 2sqrt{i+1})<infty$$
          (we use the fact that $mathbb P(M_n>x)leqslant nmathbb P(X_1gt x)$).



          We thus deduce that $limsup_iM_{2^{i}}/(sqrt{2(i+1)})leqslant 1+varepsilon$ almost surely. Taking $varepsilon:=1/k$, we get $limsup_iM_{2^{i}}/(sqrt{2(i+1)})leqslant 1$ almost surely. To conclude, notice that if $2^ileqslant nlt 2^{i+1}$,
          $$frac{M_n}{sqrt{2log n}}leqslant frac{M_{2^{i+1}}}{sqrt{2i}}.$$



          For the $liminf$, define for a fixed positive $varepsilon$, $$A_n:=left{frac{M_n}{sqrt{2log n}}lt 1-varepsilonright}.$$
          A use of the estimate on the tail of the normal distribution show that the series $sum_n mathbb P(A_n)$ is convergent, hence by the Borel-Cantelli lemma, we have $mathbb Pleft(limsup_n A_nright)=0$. This means that for almost every $omega$, we can find $N=N(omega)$ such that if $ngeqslant N(omega)$, then $$frac{M_n(omega)}{sqrt{2log n}}geqslant 1-varepsilon.$$
          This implies that $$liminf_{nto infty}frac{M_n(omega)}{sqrt{2log n}}geqslant 1-varepsilon quad mbox{a.e.}$$
          Here again, taking $varepsilon:=1/k$, we obtain that
          $$liminf_{nto infty}frac{M_n(omega)}{sqrt{2log n}}geqslant 1quad mbox{a.e.}$$






          share|cite|improve this answer











          $endgroup$













          • $begingroup$
            So I am mainly having trouble with justifying the first sum converges. I get $$mathbb{P}(M_{2^i}>sqrt{(1+epsilon)2(i+1)})=1-(1-mathbb{P}(X_{2^i}>sqrt{(1+epsilon)2(i+1)}))^{2^i} leq 1-(1-frac{1}{sqrt{(1+epsilon)2(i+1)}}e^{-(1+epsilon)(i+1)})^{2^i}$$
            $endgroup$
            – anonymous
            Oct 7 '14 at 10:15












          • $begingroup$
            I edited. Is it clearer?
            $endgroup$
            – Davide Giraudo
            Oct 7 '14 at 10:18










          • $begingroup$
            yes significantly. Thank you. So that shows that $limsup$ of $frac{M_n}{sqrt{2log n}}leq 1$, but not the $liminfgeq1$.
            $endgroup$
            – anonymous
            Oct 7 '14 at 10:27












          • $begingroup$
            It may be not the case that $liminfgeq 1$, however the limsup is greater than $1$ because $M_ngeq X_n$.
            $endgroup$
            – Davide Giraudo
            Oct 7 '14 at 11:10










          • $begingroup$
            Sorry, I missed the fact that you have a limit and not only a limsup.
            $endgroup$
            – Davide Giraudo
            Oct 7 '14 at 11:22














          6












          6








          6





          $begingroup$

          Your estimate gives that for each positive $varepsilon$, we have
          $$sum_imathbb P(M_{2^{i}}>(1+varepsilon)sqrt 2sqrt{i+1})<infty$$
          (we use the fact that $mathbb P(M_n>x)leqslant nmathbb P(X_1gt x)$).



          We thus deduce that $limsup_iM_{2^{i}}/(sqrt{2(i+1)})leqslant 1+varepsilon$ almost surely. Taking $varepsilon:=1/k$, we get $limsup_iM_{2^{i}}/(sqrt{2(i+1)})leqslant 1$ almost surely. To conclude, notice that if $2^ileqslant nlt 2^{i+1}$,
          $$frac{M_n}{sqrt{2log n}}leqslant frac{M_{2^{i+1}}}{sqrt{2i}}.$$



          For the $liminf$, define for a fixed positive $varepsilon$, $$A_n:=left{frac{M_n}{sqrt{2log n}}lt 1-varepsilonright}.$$
          A use of the estimate on the tail of the normal distribution show that the series $sum_n mathbb P(A_n)$ is convergent, hence by the Borel-Cantelli lemma, we have $mathbb Pleft(limsup_n A_nright)=0$. This means that for almost every $omega$, we can find $N=N(omega)$ such that if $ngeqslant N(omega)$, then $$frac{M_n(omega)}{sqrt{2log n}}geqslant 1-varepsilon.$$
          This implies that $$liminf_{nto infty}frac{M_n(omega)}{sqrt{2log n}}geqslant 1-varepsilon quad mbox{a.e.}$$
          Here again, taking $varepsilon:=1/k$, we obtain that
          $$liminf_{nto infty}frac{M_n(omega)}{sqrt{2log n}}geqslant 1quad mbox{a.e.}$$






          share|cite|improve this answer











          $endgroup$



          Your estimate gives that for each positive $varepsilon$, we have
          $$sum_imathbb P(M_{2^{i}}>(1+varepsilon)sqrt 2sqrt{i+1})<infty$$
          (we use the fact that $mathbb P(M_n>x)leqslant nmathbb P(X_1gt x)$).



          We thus deduce that $limsup_iM_{2^{i}}/(sqrt{2(i+1)})leqslant 1+varepsilon$ almost surely. Taking $varepsilon:=1/k$, we get $limsup_iM_{2^{i}}/(sqrt{2(i+1)})leqslant 1$ almost surely. To conclude, notice that if $2^ileqslant nlt 2^{i+1}$,
          $$frac{M_n}{sqrt{2log n}}leqslant frac{M_{2^{i+1}}}{sqrt{2i}}.$$



          For the $liminf$, define for a fixed positive $varepsilon$, $$A_n:=left{frac{M_n}{sqrt{2log n}}lt 1-varepsilonright}.$$
          A use of the estimate on the tail of the normal distribution show that the series $sum_n mathbb P(A_n)$ is convergent, hence by the Borel-Cantelli lemma, we have $mathbb Pleft(limsup_n A_nright)=0$. This means that for almost every $omega$, we can find $N=N(omega)$ such that if $ngeqslant N(omega)$, then $$frac{M_n(omega)}{sqrt{2log n}}geqslant 1-varepsilon.$$
          This implies that $$liminf_{nto infty}frac{M_n(omega)}{sqrt{2log n}}geqslant 1-varepsilon quad mbox{a.e.}$$
          Here again, taking $varepsilon:=1/k$, we obtain that
          $$liminf_{nto infty}frac{M_n(omega)}{sqrt{2log n}}geqslant 1quad mbox{a.e.}$$







          share|cite|improve this answer














          share|cite|improve this answer



          share|cite|improve this answer








          edited Oct 8 '14 at 13:36

























          answered Oct 7 '14 at 9:29









          Davide GiraudoDavide Giraudo

          128k17156268




          128k17156268












          • $begingroup$
            So I am mainly having trouble with justifying the first sum converges. I get $$mathbb{P}(M_{2^i}>sqrt{(1+epsilon)2(i+1)})=1-(1-mathbb{P}(X_{2^i}>sqrt{(1+epsilon)2(i+1)}))^{2^i} leq 1-(1-frac{1}{sqrt{(1+epsilon)2(i+1)}}e^{-(1+epsilon)(i+1)})^{2^i}$$
            $endgroup$
            – anonymous
            Oct 7 '14 at 10:15












          • $begingroup$
            I edited. Is it clearer?
            $endgroup$
            – Davide Giraudo
            Oct 7 '14 at 10:18










          • $begingroup$
            yes significantly. Thank you. So that shows that $limsup$ of $frac{M_n}{sqrt{2log n}}leq 1$, but not the $liminfgeq1$.
            $endgroup$
            – anonymous
            Oct 7 '14 at 10:27












          • $begingroup$
            It may be not the case that $liminfgeq 1$, however the limsup is greater than $1$ because $M_ngeq X_n$.
            $endgroup$
            – Davide Giraudo
            Oct 7 '14 at 11:10










          • $begingroup$
            Sorry, I missed the fact that you have a limit and not only a limsup.
            $endgroup$
            – Davide Giraudo
            Oct 7 '14 at 11:22


















          • $begingroup$
            So I am mainly having trouble with justifying the first sum converges. I get $$mathbb{P}(M_{2^i}>sqrt{(1+epsilon)2(i+1)})=1-(1-mathbb{P}(X_{2^i}>sqrt{(1+epsilon)2(i+1)}))^{2^i} leq 1-(1-frac{1}{sqrt{(1+epsilon)2(i+1)}}e^{-(1+epsilon)(i+1)})^{2^i}$$
            $endgroup$
            – anonymous
            Oct 7 '14 at 10:15












          • $begingroup$
            I edited. Is it clearer?
            $endgroup$
            – Davide Giraudo
            Oct 7 '14 at 10:18










          • $begingroup$
            yes significantly. Thank you. So that shows that $limsup$ of $frac{M_n}{sqrt{2log n}}leq 1$, but not the $liminfgeq1$.
            $endgroup$
            – anonymous
            Oct 7 '14 at 10:27












          • $begingroup$
            It may be not the case that $liminfgeq 1$, however the limsup is greater than $1$ because $M_ngeq X_n$.
            $endgroup$
            – Davide Giraudo
            Oct 7 '14 at 11:10










          • $begingroup$
            Sorry, I missed the fact that you have a limit and not only a limsup.
            $endgroup$
            – Davide Giraudo
            Oct 7 '14 at 11:22
















          $begingroup$
          So I am mainly having trouble with justifying the first sum converges. I get $$mathbb{P}(M_{2^i}>sqrt{(1+epsilon)2(i+1)})=1-(1-mathbb{P}(X_{2^i}>sqrt{(1+epsilon)2(i+1)}))^{2^i} leq 1-(1-frac{1}{sqrt{(1+epsilon)2(i+1)}}e^{-(1+epsilon)(i+1)})^{2^i}$$
          $endgroup$
          – anonymous
          Oct 7 '14 at 10:15






          $begingroup$
          So I am mainly having trouble with justifying the first sum converges. I get $$mathbb{P}(M_{2^i}>sqrt{(1+epsilon)2(i+1)})=1-(1-mathbb{P}(X_{2^i}>sqrt{(1+epsilon)2(i+1)}))^{2^i} leq 1-(1-frac{1}{sqrt{(1+epsilon)2(i+1)}}e^{-(1+epsilon)(i+1)})^{2^i}$$
          $endgroup$
          – anonymous
          Oct 7 '14 at 10:15














          $begingroup$
          I edited. Is it clearer?
          $endgroup$
          – Davide Giraudo
          Oct 7 '14 at 10:18




          $begingroup$
          I edited. Is it clearer?
          $endgroup$
          – Davide Giraudo
          Oct 7 '14 at 10:18












          $begingroup$
          yes significantly. Thank you. So that shows that $limsup$ of $frac{M_n}{sqrt{2log n}}leq 1$, but not the $liminfgeq1$.
          $endgroup$
          – anonymous
          Oct 7 '14 at 10:27






          $begingroup$
          yes significantly. Thank you. So that shows that $limsup$ of $frac{M_n}{sqrt{2log n}}leq 1$, but not the $liminfgeq1$.
          $endgroup$
          – anonymous
          Oct 7 '14 at 10:27














          $begingroup$
          It may be not the case that $liminfgeq 1$, however the limsup is greater than $1$ because $M_ngeq X_n$.
          $endgroup$
          – Davide Giraudo
          Oct 7 '14 at 11:10




          $begingroup$
          It may be not the case that $liminfgeq 1$, however the limsup is greater than $1$ because $M_ngeq X_n$.
          $endgroup$
          – Davide Giraudo
          Oct 7 '14 at 11:10












          $begingroup$
          Sorry, I missed the fact that you have a limit and not only a limsup.
          $endgroup$
          – Davide Giraudo
          Oct 7 '14 at 11:22




          $begingroup$
          Sorry, I missed the fact that you have a limit and not only a limsup.
          $endgroup$
          – Davide Giraudo
          Oct 7 '14 at 11:22











          2












          $begingroup$

          Not an answer, but a related comment that is too long for a comment box ...



          This question made me curious to compare:




          • the pdf of the sample maximum (given a sample of size $n$ drawn on a N(0,1) parent), say $f(x;n):$


          $$f(x) = frac{2^{frac{1}{2}-n} n e^{-frac{x^2}{2}} left(1+text{erf}left(frac{x}{sqrt{2}}right)right)^{n-1}}{sqrt{pi }}$$
          where erf(.) denotes the error function, to




          • the asymptote proposed by the question $sqrt{2 log n}$


          ... when $n$ is very large (say $n$ = 1 million).



          The diagram compares:



          enter image description here



          The distribution does not, in fact, converge to the asymptote in any meaningful manner, even when $n$ is very large, such as $n$ = 1 million or 100 million or even a billion.






          share|cite|improve this answer











          $endgroup$













          • $begingroup$
            Are you using the right logarithm? (I.e. base $e$ and not base 10?)
            $endgroup$
            – Chill2Macht
            Oct 1 '18 at 0:13






          • 1




            $begingroup$
            Yes - $sqrt{2 log left(10^6right)}$ = 5.25 ... as shown in the diagram above (the red line). By contrast, with base 10, the same calculation would be equal to $2 sqrt{3}$ which is approx 3.46.
            $endgroup$
            – wolfies
            Oct 1 '18 at 7:17


















          2












          $begingroup$

          Not an answer, but a related comment that is too long for a comment box ...



          This question made me curious to compare:




          • the pdf of the sample maximum (given a sample of size $n$ drawn on a N(0,1) parent), say $f(x;n):$


          $$f(x) = frac{2^{frac{1}{2}-n} n e^{-frac{x^2}{2}} left(1+text{erf}left(frac{x}{sqrt{2}}right)right)^{n-1}}{sqrt{pi }}$$
          where erf(.) denotes the error function, to




          • the asymptote proposed by the question $sqrt{2 log n}$


          ... when $n$ is very large (say $n$ = 1 million).



          The diagram compares:



          enter image description here



          The distribution does not, in fact, converge to the asymptote in any meaningful manner, even when $n$ is very large, such as $n$ = 1 million or 100 million or even a billion.






          share|cite|improve this answer











          $endgroup$













          • $begingroup$
            Are you using the right logarithm? (I.e. base $e$ and not base 10?)
            $endgroup$
            – Chill2Macht
            Oct 1 '18 at 0:13






          • 1




            $begingroup$
            Yes - $sqrt{2 log left(10^6right)}$ = 5.25 ... as shown in the diagram above (the red line). By contrast, with base 10, the same calculation would be equal to $2 sqrt{3}$ which is approx 3.46.
            $endgroup$
            – wolfies
            Oct 1 '18 at 7:17
















          2












          2








          2





          $begingroup$

          Not an answer, but a related comment that is too long for a comment box ...



          This question made me curious to compare:




          • the pdf of the sample maximum (given a sample of size $n$ drawn on a N(0,1) parent), say $f(x;n):$


          $$f(x) = frac{2^{frac{1}{2}-n} n e^{-frac{x^2}{2}} left(1+text{erf}left(frac{x}{sqrt{2}}right)right)^{n-1}}{sqrt{pi }}$$
          where erf(.) denotes the error function, to




          • the asymptote proposed by the question $sqrt{2 log n}$


          ... when $n$ is very large (say $n$ = 1 million).



          The diagram compares:



          enter image description here



          The distribution does not, in fact, converge to the asymptote in any meaningful manner, even when $n$ is very large, such as $n$ = 1 million or 100 million or even a billion.






          share|cite|improve this answer











          $endgroup$



          Not an answer, but a related comment that is too long for a comment box ...



          This question made me curious to compare:




          • the pdf of the sample maximum (given a sample of size $n$ drawn on a N(0,1) parent), say $f(x;n):$


          $$f(x) = frac{2^{frac{1}{2}-n} n e^{-frac{x^2}{2}} left(1+text{erf}left(frac{x}{sqrt{2}}right)right)^{n-1}}{sqrt{pi }}$$
          where erf(.) denotes the error function, to




          • the asymptote proposed by the question $sqrt{2 log n}$


          ... when $n$ is very large (say $n$ = 1 million).



          The diagram compares:



          enter image description here



          The distribution does not, in fact, converge to the asymptote in any meaningful manner, even when $n$ is very large, such as $n$ = 1 million or 100 million or even a billion.







          share|cite|improve this answer














          share|cite|improve this answer



          share|cite|improve this answer








          edited Oct 1 '18 at 7:20

























          answered Apr 2 '16 at 14:14









          wolfieswolfies

          4,2492923




          4,2492923












          • $begingroup$
            Are you using the right logarithm? (I.e. base $e$ and not base 10?)
            $endgroup$
            – Chill2Macht
            Oct 1 '18 at 0:13






          • 1




            $begingroup$
            Yes - $sqrt{2 log left(10^6right)}$ = 5.25 ... as shown in the diagram above (the red line). By contrast, with base 10, the same calculation would be equal to $2 sqrt{3}$ which is approx 3.46.
            $endgroup$
            – wolfies
            Oct 1 '18 at 7:17




















          • $begingroup$
            Are you using the right logarithm? (I.e. base $e$ and not base 10?)
            $endgroup$
            – Chill2Macht
            Oct 1 '18 at 0:13






          • 1




            $begingroup$
            Yes - $sqrt{2 log left(10^6right)}$ = 5.25 ... as shown in the diagram above (the red line). By contrast, with base 10, the same calculation would be equal to $2 sqrt{3}$ which is approx 3.46.
            $endgroup$
            – wolfies
            Oct 1 '18 at 7:17


















          $begingroup$
          Are you using the right logarithm? (I.e. base $e$ and not base 10?)
          $endgroup$
          – Chill2Macht
          Oct 1 '18 at 0:13




          $begingroup$
          Are you using the right logarithm? (I.e. base $e$ and not base 10?)
          $endgroup$
          – Chill2Macht
          Oct 1 '18 at 0:13




          1




          1




          $begingroup$
          Yes - $sqrt{2 log left(10^6right)}$ = 5.25 ... as shown in the diagram above (the red line). By contrast, with base 10, the same calculation would be equal to $2 sqrt{3}$ which is approx 3.46.
          $endgroup$
          – wolfies
          Oct 1 '18 at 7:17






          $begingroup$
          Yes - $sqrt{2 log left(10^6right)}$ = 5.25 ... as shown in the diagram above (the red line). By contrast, with base 10, the same calculation would be equal to $2 sqrt{3}$ which is approx 3.46.
          $endgroup$
          – wolfies
          Oct 1 '18 at 7:17




















          draft saved

          draft discarded




















































          Thanks for contributing an answer to Mathematics Stack Exchange!


          • Please be sure to answer the question. Provide details and share your research!

          But avoid



          • Asking for help, clarification, or responding to other answers.

          • Making statements based on opinion; back them up with references or personal experience.


          Use MathJax to format equations. MathJax reference.


          To learn more, see our tips on writing great answers.




          draft saved


          draft discarded














          StackExchange.ready(
          function () {
          StackExchange.openid.initPostLogin('.new-post-login', 'https%3a%2f%2fmath.stackexchange.com%2fquestions%2f961780%2fprove-that-the-maximum-of-n-independent-standard-normal-random-variables-is-a%23new-answer', 'question_page');
          }
          );

          Post as a guest















          Required, but never shown





















































          Required, but never shown














          Required, but never shown












          Required, but never shown







          Required, but never shown

































          Required, but never shown














          Required, but never shown












          Required, but never shown







          Required, but never shown







          Popular posts from this blog

          Can a sorcerer learn a 5th-level spell early by creating spell slots using the Font of Magic feature?

          Does disintegrating a polymorphed enemy still kill it after the 2018 errata?

          A Topological Invariant for $pi_3(U(n))$